LSAT and Law School Admissions Forum

Get expert LSAT preparation and law school admissions advice from PowerScore Test Preparation.

 Administrator
PowerScore Staff
  • PowerScore Staff
  • Posts: 8937
  • Joined: Feb 02, 2011
|
#91375
Complete Question Explanation

The correct answer choice is (B).

Answer choice (A):

Answer choice (B): This is the correct answer choice.

Answer choice (C):

Answer choice (D):

Answer choice (E):

This explanation is still in progress. Please post any questions below!
User avatar
 mkarimi73
  • Posts: 73
  • Joined: Aug 18, 2022
|
#97722
Could I have an explanation as to why (C) can be safely removed? Thanks.
 Rachael Wilkenfeld
PowerScore Staff
  • PowerScore Staff
  • Posts: 1419
  • Joined: Dec 15, 2011
|
#97783
Hi mkarimi73,

According to Passage A, the fact that it was short-lived did not matter. We would expect mistruths to be quickly and accurately dispelled. The market quickly adjusts to what the informed people know. The informed people would know that there is manipulation happening and would make expected adjustments as they did in this scenario. That means that the scenario supports Passage A, and does so as described in answer choice (B).

Hope that helps!
User avatar
 ashpine17
  • Posts: 322
  • Joined: Apr 06, 2021
|
#97902
My gut was telling me that this scenario supported passage a since it showed that the market was self-regulating but the issue I had with B was that the information was only among people who were well-informed, not others. so I dont understand how this is supposed to support B.
 Robert Carroll
PowerScore Staff
  • PowerScore Staff
  • Posts: 1787
  • Joined: Dec 06, 2013
|
#98274
ashpine17,

The first sentence of the third paragraph of passage A talks about exactly such a situation. Well-informed people have the knowledge and that knowledge is spread throughout the entire market by their actions. That's what we have in answer choice (B).

Robert Carroll
User avatar
 CristinaCP
  • Posts: 28
  • Joined: Sep 17, 2023
|
#105851
Hi Powerscore,

For A, I understand that the hypothetical isn't even showing the broad generalization contracts for an initially unpopular candidate cannot have a sustained increase in value; it's only showing that one example of market manipulation did not work because well-informed people can influence the market. But can we also eliminate A because Passage A doesn't even make the point that "contracts for an initially unpopular candidate cannot have a sustained increase in value?" I don't see how it could support Passage A for that reason if Passage A never makes the point, so that's part of the reason why I eliminated it.
 Luke Haqq
PowerScore Staff
  • PowerScore Staff
  • Posts: 887
  • Joined: Apr 26, 2012
|
#105872
Hi CristinaCP!

You comment,

But can we also eliminate A because Passage A doesn't even make the point that "contracts for an initially unpopular candidate cannot have a sustained increase in value?"
Yes, your reasoning provides good reasons for eliminating answer choice (A). You're right to note that Passage A never really makes a point to this end. There's nothing in that passage that implies that an unpopular candidate cannot have a sustained increase in value.

Get the most out of your LSAT Prep Plus subscription.

Analyze and track your performance with our Testing and Analytics Package.